Concentric Circles, Tangent Circles

Geometry Level 5

Circle Γ 1 \Gamma_1 with center O 1 O_1 is drawn. A smaller circle Γ 2 \Gamma_2 with center O 1 O_1 is drawn. Circle Γ 3 \Gamma_3 with center O 2 O_2 is drawn such that it is internally tangent to both circles Γ 1 \Gamma_1 and Γ 2 \Gamma_2 . The radius of circle Γ 1 \Gamma_1 that is perpendicular to O 1 O 2 \overline{O_1O_2} is drawn; it intersects Γ 1 \Gamma_1 at A A , Γ 2 \Gamma_2 at B B , and Γ 3 \Gamma_3 at P P .

Let the minimum possible value of A P A B \dfrac{AP}{AB} for any choice of circles Γ 1 \Gamma_1 and Γ 2 \Gamma_2 be m m . Find 1000 m \lfloor 1000m\rfloor


The answer is 500.

This section requires Javascript.
You are seeing this because something didn't load right. We suggest you, (a) try refreshing the page, (b) enabling javascript if it is disabled on your browser and, finally, (c) loading the non-javascript version of this page . We're sorry about the hassle.

3 solutions

Daniel Liu
Jun 10, 2014

Imgur Imgur

Draw and label as in the above diagram.

First, notice that D P C = 9 0 \angle DPC=90^{\circ} because C D CD is the diameter of circle Γ 3 \Gamma_3 and P P is on Γ 3 \Gamma_3 .

Also, we know that D O 1 A = 9 0 \angle DO_1A=90^{\circ} .

Thus, D O 1 P P O 1 C \triangle DO_1P\sim \triangle PO_1C .

Therefore the ratio of their sides are the same: D O 1 O 1 P = O 1 P O 1 C \dfrac{DO_1}{O_1P}=\dfrac{O_1P}{O_1C}

Rearranging gives ( O 1 P ) 2 = ( D O 1 ) ( O 1 C ) (O_1P)^2=(DO_1)(O_1C) or O 1 P = D O 1 O 1 C O_1P=\sqrt{DO_1\cdot O_1C}

Now, we know that D O 1 = B O 1 DO_1=BO_1 and A O 1 = C O 1 AO_1=CO_1 because they are radii of their respective circles.

Thus, O 1 P = A O 1 B O 1 O_1P=\sqrt{AO_1\cdot BO_1}

Thus, A P = A O 1 P O 1 = A O 1 A O 1 B O 1 AP=AO_1-PO_1=AO_1-\sqrt{AO_1\cdot BO_1}

Also, A B = A O 1 B O 1 AB=AO_1-BO_1 .

Thus, we need to find the minimum of A O 1 A O 1 B O 1 A O 1 B O 1 \dfrac{AO_1-\sqrt{AO_1\cdot BO_1}}{AO_1-BO_1}

By AM-GM, we have A O 1 B O 1 A O 1 + B O 1 2 -\sqrt{AO_1\cdot BO_1}\ge -\dfrac{AO_1+BO_1}{2} , so A O 1 A O 1 B O 1 A O 1 B O 1 A O 1 A O 1 + B O 1 2 A O 1 B O 1 = A O 1 B O 1 2 A O 1 B O 1 = 1 2 \dfrac{AO_1-\sqrt{AO_1\cdot BO_1}}{AO_1-BO_1}\ge \dfrac{AO_1-\dfrac{AO_1+BO_1}{2}}{AO_1-BO_1}=\dfrac{\dfrac{AO_1-BO_1}{2}}{AO_1-BO_1}=\dfrac{1}{2}

Thus, m = 1 2 m=\dfrac{1}{2} , so 1000 m = 500 \lfloor 1000m\rfloor = \boxed{500}


Note: Equality case is when A O 1 = B O 1 AO_1=BO_1 . However, technically this can never be achieved, because when A O 1 = B O 1 AO_1=BO_1 then A P A B = 0 0 \dfrac{AP}{AB}=\dfrac{0}{0} which is not defined. Thus, we actually have m > 1 2 m > \dfrac{1}{2} . However, this does not affect the answer.

Interestingly one can show that A P P B = A O 1 B O 1 \frac {AP}{PB}=\sqrt {\frac {AO_1}{BO_1}}

Xuming Liang - 7 years ago

You could just say "Altitude Theorem" rather than proving it...

Finn Hulse - 7 years ago

Log in to reply

In context of circles it's essentially just power of a point.

Xuming Liang - 7 years ago

Log in to reply

That's true. I'm just pointing out that he's reproving something. :P

Finn Hulse - 7 years ago

Log in to reply

@Finn Hulse I wanted a full solution. So I did everything.

Daniel Liu - 7 years ago

Log in to reply

@Daniel Liu Then why didn't you prove that an inscribed triangle on a semicircle is a right triangle? You just made that assertion without proof. :D

Finn Hulse - 7 years ago

Log in to reply

@Finn Hulse Because that is one of the most basic properties of angles. Power of a Point is not as trivial.

And besides, why do you care?

Daniel Liu - 7 years ago

Log in to reply

@Daniel Liu For sake of brevity.

Finn Hulse - 7 years ago

How to prove that the point O 2 O_2 is on the second circle?

mathh mathh - 6 years, 12 months ago

Log in to reply

I never used the fact that O 2 O_2 is on the second circle because that is not true. It was just a coincidence that O 2 O_2 was on the second circle in the diagram.

Daniel Liu - 6 years, 12 months ago
Nam Diện Lĩnh
Jun 11, 2014

Let O 1 O_1 be ( 0 , 0 ) (0,0) in O x y Oxy Coordinate and let O 1 A = 1 O_1A=1 , O 1 B = r ( r < O 1 A = 1 ) O_1B=r(r<O_1A=1) .

We then have:

  • R O 2 = 1 + r 2 R_{O_2}=\frac{1+r}{2}
  • Coordinate of O 2 O_2 is ( 1 r 2 , 0 ) (\frac{1-r}{2},0)
  • A B = 1 r AB=1-r
  • A P = O 1 A O 1 P = O 2 P 2 O 1 O 2 = 1 ( 1 + r 2 ) 2 ( 1 r 2 ) 2 = 1 r AP=O_1A-O_1P=\sqrt{{O_2P}^2-O_1O_2}=1-\sqrt{(\frac{1+r}{2})^2-(\frac{1-r}{2})^2}=1-\sqrt{r}

That means m = m i n ( A P A B ) = m i n ( 1 r 1 r ) = m i n ( 1 1 + r ) = 1 2 m=min(\frac{AP}{AB})=min(\frac{1-\sqrt{r}}{1-r})=min(\frac{1}{1+\sqrt{r}})=\frac{1}{2}

when r = 1 r=1

Aaaaa Bbbbb
Jun 10, 2014

Call x=AP. A P ( 2 R A P ) = R r 2 R 2 = ( R r ) R AP*(2R-AP)=\frac{R-r}{\sqrt{2}}*R*\sqrt{2}=(R-r)*R x 2 2 R x + ( R r ) R = 0 \Rightarrow x^2-2Rx+(R-r)*R=0 A P = r R + R AP=-\sqrt{rR}+R S = A P A B = R r R R r S=\frac{AP}{AB}=\frac{R-\sqrt{rR}}{R-r} S = R ( R r ) R r S=\frac{\sqrt{R}(\sqrt{R}-\sqrt{r})}{R-r} S = R R + r S=\frac{\sqrt{R}}{\sqrt{R}+\sqrt{r}} L i m r R S = 1 2 Lim_{r \rightarrow R}{S}=\frac{1}{2} Because when r increases then S is decreases, but max(r)=R, at this r: S=1/2. 1000 S = 500 \lfloor 1000S \rfloor = \boxed{500}

Have you proved that the minimum happens when r R r\to R ?

Daniel Liu - 7 years ago

Log in to reply

S decreases when r increases, so minimum of S happens when r gets to its maximum, that is R

Nam Diện Lĩnh - 7 years ago

Log in to reply

Good! ¨ \ddot\smile

Daniel Liu - 7 years ago

0 pending reports

×

Problem Loading...

Note Loading...

Set Loading...